You are on page 1of 44

High-speed passenger trains such

as the one shown here are used in


many countries, including Japan,
France, England, Germany, and
South Korea. These trains have
operational speeds from 200 to
300 km/h. A train moving along a
straight track is an example of
one-dimensional motion. The train
in the diagram is covering greater
distances in equal time intervals—
in other words, it is accelerating.

(c) ©Robert Harding World Imagery/Alamy Photos

34
CHAPTER 2

Motion
Section 1
Displacement and
Velocity
Section 2

in One
Acceleration
Section 3
Falling Objects

Dimension Why It Matters


Velocity and acceleration
are involved in many
aspects of everyday life,
from riding a bicycle to
driving a car to traveling
on a high-speed train.
The definitions and
equations you will study
in this chapter allow you
to make predictions
about these aspects of
motion, given certain
initial conditions.

Online Physics
HMDScience.com

Online Labs
Motion
Premium
Acceleration Content

Free-Fall Acceleration Physics


HMDScience.com
Free Fall
Acceleration in One Dimension

35
Section 1
Objectives
Describe motion in terms of Displacement
and Velocity
frame of reference,
displacement, time, and
velocity.

Calculate the displacement of


an object traveling at a known Key Terms
frame of reference average velocity
velocity for a specific time
displacement instantaneous velocity
interval.

Construct and interpret graphs


of position versus time.
Motion
Motion happens all around us. Every day, we see objects such as cars,
people, and soccer balls move in different directions with different
speeds. We are so familiar with the idea of motion that it requires a
special effort to analyze motion as a physicist does.

One-dimensional motion is the simplest form of motion.


One way to simplify the concept of motion is to consider only the kinds
of motion that take place in one direction. An example of this one-
dimensional motion is the motion of a commuter train on a straight track,
as in Figure 1.1.
In this one-dimensional motion, the train can move either forward or
backward along the tracks. It cannot move left and right or up and down.
This chapter deals only with one-dimensional motion. In later chapters,
frame of reference  a system for
specifying the precise location of you will learn how to describe more complicated motions such as the
objects in space and time motion of thrown baseballs and other projectiles.

Figure 1.1 Motion takes place over time and depends upon the
frame of reference.
Frames of Reference  The motion of a commuter
train traveling along a straight route is an example of It seems simple to describe the motion of the train. As the
one-dimensional motion. Each train can move only train in Figure 1.1 begins its route, it is at the first station. Later,
forward and backward along the tracks. it will be at another station farther down the tracks. But Earth
is spinning on its axis, so the train, stations, and the tracks are
also moving around the axis. At the same time, Earth is
moving around the sun. The sun and the rest of the solar
system are moving through our galaxy. This galaxy is traveling
©Courtesy of the New York Transit Museum, Brooklyn

through space as well.


When faced with a complex situation like this, physicists
break it down into simpler parts. One key approach is to
choose a frame of reference against which you can measure
changes in position. In the case of the train, any of the stations
along its route could serve as a convenient frame of reference.
When you select a reference frame, note that it remains fixed
for the problem in question and has an origin, or starting
point, from which the motion is measured.

36 Chapter 2
If an object is at rest (not moving), its position does not change
with respect to a fixed frame of reference. For example, the benches on Conceptual Challenge
the platform of one subway station never move down the tracks to
Space Shuttle  A space shuttle
another station.
takes off from Florida and
In physics, any frame of reference can be chosen as long as it is used circles Earth several times,
consistently. If you are consistent, you will get the same results, no matter finally landing in California. While
which frame of reference you choose. But some frames of reference can the shuttle is in flight, a pho-
make explaining things easier than other frames of reference. tographer flies from Florida to
California to take pictures of the
For example, when considering the motion of the gecko in Figure 1.2,
astronauts when they step off
it is useful to imagine a stick marked in centimeters placed under the
the shuttle. Who undergoes
gecko’s feet to define the frame of reference. The measuring stick serves
the greater displacement,
as an x-axis. You can use it to identify the gecko’s initial position and its
the photographer or the
final position.
astronauts?
Roundtrip  What is the
difference be­tween
Displacement the displacement of
As any object moves from one position to another, the length of the the photographer
straight line drawn from its initial position to the object’s final position is flying from Florida
called the displacement of the object. to California and the
displacement of the
astronauts flying from
Displacement is a change in position. California back to
The gecko in Figure 1.2 moves from left to right along the x-axis from an Florida?
initial position, xi , to a final position, xf . The gecko’s displacement is the
difference between its final and initial coordinates, or xf − xi . In this case,
the displacement is about 61 cm (85 cm − 24 cm). The Greek letter delta displacement  the change in position
(∆) before the x denotes a change in the position of an object. of an object

Tips and Tricks


Displacement
A change in any quantity, indicated
∆x = xf − xi by the Greek symbol delta (∆),
is equal to the final value minus
the initial value. When calculating
displacement = change in position = final position − initial position displacement, always be sure to
subtract the initial position from the
final position so that your answer has
Figure 1.2 the correct sign.

Measuring Displacement
A gecko moving along the x-axis from xi to xf undergoes a
displacement of ∆x = xf − xi.

∆x
(bl) ©Space Frontiers/Taxi/Getty Images

0 10 20 30 40 50 60 70 80 90

xi xf

Motion in One Dimension 37


Figure 1.3 Now suppose the gecko runs up a tree, as shown in Figure 1.3. In this
case, we place the measuring stick parallel to the tree. The measuring
Comparing Displacement and stick can serve as the y-axis of our coordinate system. The gecko’s initial
Distance  When the gecko is climbing and final positions are indicated by yi and yf , respectively, and the gecko’s
a tree, the displacement is measured on displacement is denoted as ∆y .
the y-axis. Again, the gecko’s position is
determined by the position of the same
point on its body. Displacement is not always equal to the distance traveled.
Displacement does not always tell you the distance an object has moved.
90

For example, what if the gecko in Figure 1.3 runs up the tree from the
20 cm marker (its initial position) to the 80 cm marker. After that, it
80

yf retreats down the tree to the 50 cm marker (its final position). It has
traveled a total distance of 90 cm. However, its displacement is only 30 cm
70

(yf − yi = 50 cm − 20 cm = 30 cm). If the gecko were to return to its


starting point, its displacement would be zero because its initial position
60

and final position would be the same.


50

∆y Displacement can be positive or negative.


Displacement also includes a description of the direction of motion. In
40

one-dimensional motion, there are only two directions in which an object


can move, and these directions can be described as positive or negative.
30

In this book, unless otherwise stated, the right (or east) will be consid-
ered the positive direction and the left (or west) will be considered the
20

yi
negative direction. Similarly, upward (or north) will be considered
positive, and downward (or south) will be considered negative. Figure 1.4
10

gives examples of determining displacements for a variety of situations.


0

Figure 1.4
positive displacement Negative displacement

xi xf xf xi
∆x = xf − xi = 80 cm − 10 cm = +70 cm ∆x = xf − xi = 20 cm − 80 cm = −60 cm
PHYSICS PHYSICS
Spec. Number PH 99 PE C02-001-007-A Spec. Number PH 99 PE C02-001
Boston Graphics, Inc. Boston Graphics, Inc.
617.523.1333 617.523.1333

xi xf xf xi
∆x = xf − xi = 12 cm − 3 cm = +9 cm ∆x = xf − xi = 0 cm − 15 cm = −15 cm
PHYSICS PHYSICS
Spec. Number PH 99 PE C02-001-009-A Spec. Number PH 99 PE C02
Boston Graphics, Inc. Boston Graphics, Inc.
617.523.1333 617.523.1333
10

xi xf xf xi
∆x = xf − xi = 6 cm − (− 10 cm) = +16 cm ∆x = xf − xi = −20 cm − (− 10 cm) = −10 cm
PHYSICS PHYSICS
Spec. Number PH 99 PE C02-001-011-A Spec. Number PH 99 PE C02-001-012
38 Chapter 2 Boston Graphics, Inc. Boston Graphics, Inc.
617.523.1333 617.523.1333
Velocity
Where an object started and where it stopped does not completely
Did YOU Know?
describe the motion of the object. For example, the ground that you’re
standing on may move 8.0 cm to the left. This motion could take several The branch of physics concerned with
years and be a sign of the normal slow movement of Earth’s tectonic motion and forces is called mechanics.
The subset of mechanics that
plates. If this motion takes place in just a second, however, you may be
describes motion without regard to its
experiencing an earthquake or a landslide. Knowing the speed is impor- causes is called kinematics.
tant when evaluating motion.

Average velocity is displacement divided by the time interval.


Consider the car in Figure 1.5. The car is moving along a highway in a
straight line (the x-axis). Suppose that the positions of the car are xi at
time ti and xf at time tf . In the time interval ∆t = tf − ti , the displacement
of the car is ∆x = xf − xi . The average velocity, vavg , is defined as the average velocity  the total displace-
displacement divided by the time interval during which the displacement ment divided by the time interval during
which the displacement occurred
occurred. In SI, the unit of velocity is meters per second, abbreviated
as m/s.

Average Velocity xf − x i
​ ∆x ​ = ​ _ ​ 
vavg = _  
∆t tf − ti Tips and Tricks
change in position __ displacement Average velocity is not always equal
average velocity = __
​    
    ​= ​     ​
   to the average of the initial and final
change in time time interval velocities. For instance, if you drive
first at 40 km/h west and later at 60
km/h west, your average velocity is
The average velocity of an object can be positive or negative, depend- not necessarily 50 km/h west.
ing on the sign of the displacement. (The time interval is always positive.)
As an example, consider a car trip to a friend’s house 370 km to the west
(the negative direction) along a straight highway. If you left your house at
Figure 1.5
10 a.m. and arrived at your friend’s house at 3 p.m., your average velocity
would be as follows: Average Velocity  The average
velocity of this car tells you how fast
​ ∆x ​ = _
vavg = _ ​ −370 km = −74 km/h = 74 km/h west
 ​ 
  and in which direction it is moving.
∆t 5.0 h
ti
This value is an average. You probably did not travel exactly 74 km/h
at every moment. You may have stopped to buy gas or have lunch.
At other times, you may have traveled more slowly as a result of heavy
traffic. To make up for such delays, when you were traveling slower than I
Xi
74 km/h, there must also have been other times when you traveled faster
than 74 km/h. tf

The average velocity is equal to the constant velocity needed to cover


the given displacement in a given time interval. In the example above, if
I
you left your house and maintained a velocity of 74 km/h to the west at Xf
every moment, it would take you 5.0 h to travel 370 km.

Motion in One Dimension 39


Average Velocity and Displacement
Sample Problem A  During a race on level ground, Andrea
runs with an average velocity of 6.02 m/s to the east. What is
Andrea’s displacement after 137 s?

Analyze Given: vavg = 6.02 m/s


∆t = 137 s
Unknown: ∆x = ?

Tips and Tricks


The calculator answer is
824.74 m, but both the values
for velocity and time have
three significant figures, so
the displacement must be
reported as 825 m.
solve Rearrange the average velocity
equation to solve for displacement.
​ ∆x ​ 
vavg = _
∆t
∆x = vavg ∆t

∆x = vavg ∆t = (6.02 m/s)(137 s) =  825 m to the east

1.  Heather and Matthew walk with an average velocity of 0.98 m/s eastward. If it
takes them 34 min to walk to the store, what is their displacement?

2.  If Joe rides his bicycle in a straight line for 15 min with an average veloc­ity of
12.5 km/h south, how far has he ridden?

3.  It takes you 9.5 min to walk with an average velocity of 1.2 m/s to the north from
the bus stop to the museum entrance. What is your displacement?

4.  Simpson drives his car with an average velocity of 48.0 km/h to the east. How long
will it take him to drive 144 km on a straight highway?

5.  Look back at item 4. How much time would Simpson save by increasing his
average velocity to 56.0 km/h to the east?

6.  A bus travels 280 km south along a straight path with an average velocity of 88
km/h to the south. The bus stops for 24 min. Then, it travels 210 km south with an
average velocity of 75 km/h to the south.
a.  How long does the total trip last?
b.  What is the average velocity for the total trip?

40 Chapter 2
Velocity is not the same as speed.
In everyday language, the terms speed and velocity are used interchange-
ably. In physics, however, there is an important distinction between these
two terms. As we have seen, velocity describes motion with both a
direction and a numerical value (a magnitude) indicating how fast
something moves. However, speed has no direction, only magnitude.
An object’s average speed is equal to the distance traveled divided by
the time interval for the motion.
average speed = __​ distance traveled
  
    ​
time of travel
Figure 1.6
Velocity can be interpreted graphically.
Position-Time Graph  The motion of
The velocity of an object can be determined if the object’s position is an object moving with constant velocity will
known at specific times along its path. One way to determine this is to provide a straight-line graph of position
make a graph of the motion. Figure 1.6 represents such a graph. Notice versus time. The slope of this graph
that time is plotted on the horizontal axis and position is plotted on the indicates the velocity.
vertical axis.
The object moves 4.0 m in the time interval between t = 0 s and Position versus Time of an
t = 4.0 s. Likewise, the object moves an additional 4.0 m in the time Object at Constant Velocity
interval between t = 4.0 s and t = 8.0 s. From these data, we see that the
average velocity for each of these time intervals is +1.0 m/s (because 16.0
vavg = ∆x/∆t = 4.0 m/4.0 s). Because the average velocity does not
12.0

Position (m)
change, the object is moving with a constant velocity of +1.0 m/s, and its
motion is represented by a straight line on the position-time graph. 8.0
For any position-time graph, we can also determine the average
velocity by drawing a straight line between any two points on the graph. 4.0
The slope of this line indicates the average velocity between the positions
0
and times represented by these points. To better understand this concept, 0 2.0 4.0 6.0 8.0
compare the equation for the slope of the line with the equation for the Time (s)
average velocity.
Slope of a Line Average Velocity

change in vertical coordinates xf - xi


​ rise
slope = _ ____
run ​  = ​ change in horizontal
   
    ​ ​ ∆x ​  = ​ _ ​ 
vavg = _  
coordinates ∆t tf - ti

Conceptual Challenge
Book on a Table  A book is moved Travel  Car A travels from New York
once around the edge of a tabletop to Miami at a speed of 25 m/s.
with dimensions 1.75 m × 2.25 m. Car B travels from New York to
If the book ends up at its initial Chicago, also at a speed of 25 m/s.
position, what is its displacement? If Are the velocities of the cars equal?
it completes its motion in 23 s, what Explain.
is its average velocity? What is its
average speed?

Motion in One Dimension 41


Figure 1.7 Figure 1.7 represents straight-line graphs of position versus time for
three different objects. Object 1 has a constant positive velocity because
Position-Time Graphs  These its position increases uniformly with time. Thus, the slope of this line is
position-versus-time graphs show that positive. Object 2 has zero velocity because its position does not change
Object 1 moves with a constant positive (the object is at rest). Hence, the slope of this line is zero. Object 3 has a
velocity. Object 2 is at rest. Object 3 moves
constant negative velocity because its position decreases with time. As a
with a constant negative velocity.
result, the slope of this line is negative.

Position versus Time


of Three Objects Instantaneous velocity may not be the same as average velocity.
Now consider an object whose position-versus-time graph is not a
straight line, but a curve, as in Figure 1.8. The object moves through larger
Object 1 and larger displacements as each second passes. Thus, its velocity
increases with time.
For example, between t = 0 s and t = 2.0 s, the object moves 8.0 m,
Position

and its average velocity in this time interval is 4.0 m/s (because
Object 2 vavg = 8.0 m/2.0 s). However, between t = 0 s and t = 4.0 s, it moves
32 m, so its average velocity in this time interval is 8.0 m/s (because
vavg = 32 m/4.0 s). We obtain different average velocities, depending on
Object 3
the time interval we choose. But how can we find the velocity at an
instant of time?

Time
To determine the velocity at some instant, such as t = 3.0 s, we study a
small time interval near that instant. As the intervals become smaller and
smaller, the average velocity over that interval approaches the exact
instantaneous velocity  the velocity velocity at t = 3.0 s. This is called the instantaneous velocity. One way to
of an object at some instant or at a determine the instantaneous velocity is to construct a straight line that is
specific point in the object’s path
tangent to the position-versus-time graph at that instant. The slope of this
tangent line is equal to the value of the instantaneous velocity at that
point. For example, the instantaneous velocity of the object in Figure 1.8 at
t = 3.0 s is 12 m/s. The table lists the instantaneous velocities of the object
described by the graph in Figure 1.8. You can verify some of these values
by measuring the slope of the curve.

Figure 1.8 Position versus Time of an Object


Showing Instantaneous Velocity
Finding Instantaneous Velocity 
The instantaneous velocity at a given time
can be determined by measuring the slope
of the line that is tangent to that point on the 30
position-versus-time graph.
Position (m)

Velocity-Time Data 20

t (s) v (m/s) v = 12 m/s

0.0 0.0 10
1.0 4.0
2.0 8.0
0
3.0 12.0 0 1.0 2.0 3.0 4.0
Time (s)
4.0 16.0

42 Chapter 2
Section 1 Formative ASSESSMENT
Reviewing Main Ideas
1. What is the shortest possible time in which a bacterium could travel a
distance of 8.4 cm across a Petri dish at a constant speed of 3.5 mm/s?
2. A child is pushing a shopping cart at a speed of 1.5 m/s. How long will it
take this child to push the cart down an aisle with a length of 9.3 m?
3. An athlete swims from the north end to the south end of a 50.0 m pool in
20.0 s and makes the return trip to the starting position in 22.0 s.
a. What is the average velocity for the first half of the swim?
b. What is the average velocity for the second half of the swim?
c. What is the average velocity for the roundtrip?
4. Two students walk in the same direction along a straight path, at a con-
stant speed—one at 0.90 m/s and the other at 1.90 m/s.
a. Assuming that they start at the same point and the same time, how
much sooner does the faster student arrive at a destination 780 m
away?
b. How far would the students have to walk so that the faster student
arrives 5.50 min before the slower student?

Critical Thinking
5. Does knowing the distance between two objects give you enough infor-
mation to locate the objects? Explain.

Interpreting Graphics
6. Figure 1.9 shows position-time graphs of the straight-line movement of
two brown bears in a wildlife preserve. Which bear has the greater aver-
age velocity over the entire period? Which bear has the greater velocity
at t = 8.0 min? Is the velocity of bear A always positive? Is the velocity of
bear B ever negative?

Figure 1.9

Bear A Bear B
3000 3000

2500 2500

2000 2000
Position (m)

Position (m)

1500 1500

1000 1000
500 500
0 0
0 10 20 30 40 50 60 0 10 20 30 40 50 60
Time (min) Time (min)

TSI Graphics
HRW • Holt Physics
PH99PE-C02-001-019-A
Motion in One Dimension 43
DTSI Graphics
HRW • Holt Physics
PH99PE-C02-001-018-A
Section 2
Objectives
Describe motion in terms
changing velocity.
Acceleration
Compare graphical Key Term
representations of accelerated acceleration
and nonaccelerated motions.

Apply kinematic equations to Changes in Velocity


calculate distance, time, or Many bullet trains have a top speed of about 300 km/h. Because a train
velocity under conditions of stops to load and unload passengers, it does not always travel at that top
constant acceleration. speed. For some of the time the train is in motion, its velocity is either
increasing or decreasing. It loses speed as it slows down to stop and gains
speed as it pulls away and heads for the next station.

Acceleration is the rate of change of velocity with respect to time.


Similarly, when a shuttle bus approaches a stop, the driver begins to
apply the brakes to slow down 5.0 s before actually reaching the stop.
The speed changes from 9.0 m/s to 0 m/s over a time interval of 5.0 s.
Sometimes, however, the shuttle stops much more quickly. For example,
if the driver slams on the brakes to avoid hitting a dog, the bus slows from
9.0 m/s to 0 m/s in just 1.5 s.
Clearly, these two stops are very different, even though the shuttle’s
velocity changes by the same amount in both cases. What is different in
these two examples is the time interval during which the change in
velocity occurs. As you can imagine, this difference has a great effect
on the motion of the bus, as well as on the comfort and safety of the
passengers. A sudden change in velocity feels very different from a
slow, gradual change.
The quantity that describes the rate of change of velocity in a given
acceleration  the rate at which time interval is called acceleration. The magnitude of the average accel-
velocity changes over time; an object eration is calculated by dividing the total change in an object’s velocity by
accelerates if its speed, direction, or
both change the time interval in which the change occurs.

Average Acceleration vf - v i
​ ∆v ​ = ​ _ ​ 
aavg = _
∆t tf - ti
change in velocity
average acceleration = ___
​         ​
time required for change

Acceleration has dimensions of length divided by time squared. The


units of acceleration in SI are meters per second per second, which is
written as meters per second squared, as shown below. When measured
in these units, acceleration describes how much the velocity changes in
each second.
(m/s) _
​ _  = ​ m
s ​ 
_1 _ m
s ​  × ​ s ​  = ​ s2 ​  

44 Chapter 2
Premium Content

Average Acceleration Interactive Demo


HMDScience.com
Sample Problem B  A shuttle bus slows down with an average
acceleration of −1.8 m/s2. How long does it take the bus to slow
from 9.0 m/s to a complete stop?

Analyze Given: vi = 9.0 m/s

vf = 0 m/s

aavg = −1.8 m/s2

Unknown: ∆t = ?

solve Rearrange the average acceleration equation to solve for the time
interval.
​ ∆v ​ 
aavg = _
∆t
vf - vi 0 m/s - 9.0 m/s
​ a∆v  ​ = ​ _
∆t = _ __
aavg ​ = ​  −1.8 m/s
  
  
2
 ​
avg

∆t = 5.0 m/s

1.  As the shuttle bus comes to a sudden stop to avoid hitting a dog, it accelerates
uniformly at −4.1 m/s2 as it slows from 9.0 m/s to 0.0 m/s. Find the time interval of
acceleration for the bus.

2.  A car traveling at 7.0 m/s accelerates uniformly at 2.5 m/s2 to reach a speed of
12.0 m/s. How long does it take for this acceleration to occur?

3.  With an average acceleration of −1.2 m/s2, how long will it take a cyclist to bring a
bicycle with an initial speed of 6.5 m/s to a complete stop?

4.  Turner’s treadmill runs with a velocity of −1.2 m/s and speeds up at regular
intervals during a half-hour workout. After 25 min, the treadmill has a velocity of
−6.5 m/s. What is the average acceleration of the treadmill during this period?

5.  Suppose a treadmill has an average acceleration of 4.7 × 10−3 m/s2.


a.  How much does its speed change after 5.0 min?
b.  If the treadmill’s initial speed is 1.7 m/s, what will its final speed be?

Motion in One Dimension 45


Figure 2.1 Acceleration has direction and magnitude.
Figure 2.1 shows a high-speed train leaving a station. Imagine that the
High-Speed Train  High-
train is moving to the right so that the displacement and the velocity are
speed trains such as this one can
travel at speeds of about 300 km/h positive. The velocity increases in magnitude as the train picks up speed.
(186 mi/h). Therefore, the final velocity will be greater than the initial velocity, and
∆v will be positive. When ∆v is positive, the acceleration is positive.
On long trips with no stops, the train may travel for a while at
a constant velocity. In this situation, because the velocity is not
changing, ∆v = 0 m/s. When the velocity is constant, the
acceleration is equal to zero.
Imagine that the train, still traveling in the positive direction, slows
down as it approaches the next station. In this case, the velocity is still
positive, but the initial velocity is larger than the final velocity, so ∆v will
be negative. When ∆v is negative, the acceleration is negative.

The slope and shape of the graph describe the object’s motion.
As with all motion graphs, the slope and shape of the velocity-time graph
in Figure 2.2 allow a detailed analysis of the train’s motion over time.
When the train leaves the station, its speed is increasing over time.
The line on the graph plotting this motion slopes up and to the right,
as at point A on the graph.
When the train moves with a constant velocity, the line on the graph
continues to the right, but it is horizontal, with a slope equal to zero. This
indicates that the train’s velocity is constant, as at point B on the graph.
Finally, as the train approaches the station, its velocity decreases over
Conceptual Challenge time. The graph segment representing this motion slopes down to the
right, as at point C on the graph. This downward slope indicates that the
Fly Ball  If a baseball has zero
velocity at some instant, is the
velocity is decreasing over time.
acceleration of the baseball A negative value for the acceleration does not always indicate a
necessarily zero at that instant? decrease in speed. For example, if the train were moving in the negative
Explain, and give examples. direction, the acceleration would be negative when the train gained
Runaway Train  If a passenger speed to leave a station and positive when the train lost speed to
train is traveling on a straight enter a station.
track with a negative velocity
and a positive acceleration, is it
Figure 2.2
(tl) ©Craig Lovell/Corbis; (br) ©Aaron Haupt/Photo Researchers, Inc
speeding up or slowing down? Velocity-Time Graph
Velocity-Time Graphs  When of a Train
Hike-and-Bike Trail  When
Jennifer is out for a the velocity in the positive direction is
ride, she slows down increasing, the acceleration is positive,
on her bike as she as at point A. When the velocity is B
A
constant, there is no acceleration, as
Velocity

approach­es a group
of hikers on a trail. at point B. When the velocity in the C

Explain how her positive direction is decreasing, the


acceleration can be acceleration is negative, as at point C.
Time
positive even
though her speed
is decreasing. TSI Graphics
HRW • Holt Physics
PH99PE-C02-002-002-A

46 Chapter 2
Figure 2.3 shows how the signs of the velocity and acceleration can be
combined to give a description of an object’s motion. From this table, you
can see that a negative acceleration can describe an object that is speeding
up (when the velocity is negative) or an object that is slowing down (when
the velocity is positive). Use this table to check your answers to problems
involving acceleration.
For example, in Figure 2.2 the initial velocity vi of the train is positive.
At point A on the graph, the train’s velocity is still increasing, so its
acceleration is positive as well. The first entry in Figure 2.3 shows that in
this situation, the train is speeding up. At point C, the velocity is still
positive, but it is decreasing, so the train’s acceleration is negative.
Figure 2.4
Figure 2.3 tells you that in this case, the train is slowing down.
Motion of a Falling Ball 
Figure 2.3 The motion in this ­picture took place
signs of Velocity and Acceleration in about 1.00 s. In this short time
interval, your eyes could only detect
vi a Motion a blur. This photo shows what really
happens within that time.
+ + speeding up

- - speeding up

+ - slowing down

- + slowing down

- or + 0 constant velocity

0 - or + speeding up from rest

0 0 remaining at rest

Motion with constant acceleration.


Figure 2.4 is a strobe photograph of a ball moving in a straight line with
constant acceleration. While the ball was moving, its image was captured
ten times in one second, so the time interval between successive images
is 0.10 s. As the ball’s velocity increases, the ball travels a greater distance
during each time interval. In this example, the velocity increases by
exactly the same amount during each time interval. Thus, the accelera-
©Richard Megna/Fundamental Photographs, New York

tion is constant. Because the velocity increases for each time interval, the
successive change in displacement for each time interval increases. You
can see this in the photograph by noting that the distance between
images increases while the time interval between images remains con-
stant. The relationships between displacement, velocity, and constant
acceleration are expressed by equations that apply to any object moving
with constant acceleration.

Motion in One Dimension 47


Figure 2.5
Velocity versus Time of a Falling Ball
Constant Acceleration and
120
Average Velocity  If a ball moved for the 110
same time with a constant velocity equal to 100
vavg, it would have the same displacement 90
as the ball in Figure 2.4 moving with 80

Velocity (cm/s)
constant acceleration. 70
60
50
40
30
20
10
0
0.00 0.10 0.20 0.30 0.40 0.50 0.60 0.70 0.80 0.90 1.00
Time (s)

Displacement depends on acceleration,


TSI Graphics initial velocity, and time.
HRW • Holt Physics
Figure 2.5 is a graph of thePH99PE-C02-002-009-A
ball’s velocity plotted against time. The initial,
final, and average velocities are marked on the graph. We know that the
average velocity is equal to displacement divided by the time interval.

​ ∆x ​  
vavg = _
∆t
For an object moving with constant acceleration, the average velocity
is equal to the average of the initial velocity and the final velocity.
v i + vf   initial velocity + final velocity
Did YOU Know? vavg = ​ _  
 ​   
  average velocity = ​ ___      ​   
2 2
Decreases in speed are sometimes To find an expression for the displacement in terms of the initial and
called decelerations. Despite the sound final velocity, we can set the expressions for average velocity equal to
of the name, decelerations are really a
each other.
special case of acceleration in which v i + vf
the magnitude of the velocity—and ∆x ​  = v = _
​ _ ​   ​  
∆t avg 2
thus the speed—decreases with time.
displacement ___
__ initial velocity + final velocity
​     ​= ​ 
      
 ​   
time interval 2
Multiplying both sides of the equation by ∆t gives us an expression for
the displacement as a function of time. This equation can be used to find
the displacement of any object moving with constant acceleration.

Displacement with Constant Acceleration

∆x = ​ __12 ​(vi + vf )∆t

displacement = __
​ 12  ​(initial velocity + final velocity)(time interval)

48 Chapter 2
Premium Content

Displacement with Constant Acceleration Interactive Demo


HMDScience.com

Sample Problem C  A racing car reaches a speed of 42 m/s.


It then begins a uniform negative acceleration, using its
parachute and braking system, and comes to rest 5.5 s later.
Find the distance that the car travels during braking.

Analyze Given: vi = 42 m/s


vf = 0 m/s
∆t = 5.5 s
Unknown: ∆x = ?

Tips and Tricks


Remember that this equation
solve Use the equation that relates displacement, applies only when acceleration
initial and final velocities, and the is constant. In this problem, you
know that acceleration is constant
time interval. by the phrase “uniform negative
​ 1 ​   (vi + vf )∆t
∆x = _ acceleration.” All of the kinematic
equations introduced in this
2
chapter are valid only for constant
∆x = ​ 1 ​   (42 m/s + 0 m/s)(5.5 s)
_ acceleration.
2
∆x = 120 m

Calculator Solution
The calculator answer is 115.5. However, the velocity
and time values have only two significant figures
each, so the answer must be reported as 120 m.

1.  A car accelerates uniformly from rest to a speed of 6.6 m/s in 6.5 s. Find the
distance the car travels during this time.

2.  When Maggie applies the brakes of her car, the car slows uniformly from 15.0 m/s
to 0.0 m/s in 2.50 s. How many meters before a stop sign must she apply her brakes
in order to stop at the sign?

3.  A driver in a car traveling at a speed of 21.8 m/s sees a cat 101 m away on the road.
How long will it take for the car to accelerate uniformly to a stop in exactly 99 m?

4.  A car enters the freeway with a speed of 6.4 m/s and accelerates uniformly for
3.2 km in 3.5 min. How fast (in m/s) is the car moving after this time?

Motion in One Dimension 49


Final velocity depends on initial velocity, acceleration, and time.
What if the final velocity of the ball is not known but we still want to
calculate the displacement? If we know the initial velocity, the acceleration,
and the elapsed time, we can find the final velocity. We can then use this
value for the final velocity to find the total displacement of the ball.
By rearranging the equation for acceleration, we can find a value for
the final velocity.
∆v ​  = _vf - v i
a = ​ _ ​   ​   
∆t ∆t
a∆t = vf - vi
By adding the initial velocity to both sides of the equation, we get an
equation for the final velocity of the ball.
a∆t + vi = vf

Velocity with Constant Acceleration

vf = vi + a∆t

final velocity = initial velocity + (acceleration × time interval)

You can use this equation to find the final velocity of an object after it
has accelerated at a constant rate for any time interval.
If you want to know the displacement of an object moving with
constant acceleration over some certain time interval, you can obtain
another useful expression for displacement by substituting the expression
for vf into the expression for ∆x.
∆x = __
​ 12 ​(vi + vf)∆t

∆x = __
​ 12 ​(vi + vi + a∆t)∆t

∆x = __
​ 12 ​[2vi ∆t + a(∆t)2]

Displacement with Constant Acceleration

∆x = vi ∆t + __
​ 12 ​a(∆t)2

displacement = (initial velocity × time interval) +


__​ 1  ​acceleration × (time interval)2
2

This equation is useful not only for finding the displacement of an object
moving with constant acceleration but also for finding the displacement
required for an object to reach a certain speed or to come to a stop. For the
latter situation, you need to use both this equation and the equation
given above.

50 Chapter 2
Premium Content

Velocity and Displacement with Interactive Demo


HMDScience.com
Constant Acceleration
Sample Problem D  A plane starting at rest at one end of a
runway undergoes a uniform acceleration of 4.8 m/s2 for 15 s
before takeoff. What is its speed at takeoff? How long must the
runway be for the plane to be able to take off?

Analyze Given: vi = 0 m/s


a = 4.8 m/s2
∆t = 15 s
Unknown: vf = ?
∆x = ?

solve First, use the equation for the velocity


of a uniformly accelerated object.
vf = vi + a∆t Tips and Tricks
Because you now know vf , you
vf = 0 m/s + (4.8 m/s2)(15 s) could also use the equation
∆x =​ __12 ​( vi + vf)(∆t), or
vf = 72 m/s
∆x =​ __12 ​( 72 m/s)(15 s) = 540 m.

Then, use the displacement equation


that contains the given variables.

​ 1 ​ a(∆t)2
∆x = vi∆t + _
2
​ 1 ​ (4.8 m/s2)(15 s)2
∆x = (0 m/s)(15 s) + _
2
∆x = 540 m

1.  A car with an initial speed of 6.5 m/s accelerates at a uniform rate of
0.92 m/s2 for 3.6 s. Find the final speed and the displacement of the car during
this time.
2.  An automobile with an initial speed of 4.30 m/s accelerates uniformly at the rate
of 3.00 m/s2. Find the final speed and the displacement after 5.00 s.
3.  A car starts from rest and travels for 5.0 s with a constant acceleration of
−1.5 m/s2. What is the final velocity of the car? How far does the car travel
in this time interval?
4.  A driver of a car traveling at 15.0 m/s applies the brakes, causing a uniform
acceleration of −2.0 m/s2. How long does it take the car to accelerate to a final
speed of 10.0 m/s? How far has the car moved during the braking period?

Motion in One Dimension 51


Final velocity depends on initial velocity, acceleration,
and displacement.
So far, all of the equations for motion under uniform acceleration have
required knowing the time interval. We can also obtain an expression that
relates displacement, velocity, and acceleration without using the time
interval. This method involves rearranging one equation to solve for ∆t
and substituting that expression in another equation, making it possible
to find the final velocity of a uniformly accelerated object without know-
ing how long it has been accelerating. Start with the following equation
Did YOU Know?
for displacement:
The word physics comes from the
∆x = ​ __12 ​ (vi + vf )∆t   Now multiply both sides by 2.
ancient Greek word for “nature.”
According to Aristotle, who assigned
2∆x = (vi + vf )∆t  Next, divide both sides by (vi + vf )
the name, physics is the study of
natural events. Aristotle believed that to solve for ∆t.

( )
the study of motion was the basis
of physics. Galileo developed the ​  2∆x  ​  ​= ∆t
​_
v i + vf
foundations for the modern study of
motion using mathematics. In 1632, Now that we have an expression for ∆t, we can substitute this
Galileo published the first mathematical
expression into the equation for the final velocity.
treatment of motion.
vf = vi + a(∆t)

( )
​  2∆x  ​  ​
vf = vi + a​ _
v i + vf

In its present form, this equation is not very helpful because vf


appears on both sides. To solve for vf , first subtract vi from both sides of
the equation.

( )
​  2∆x  ​  ​
vf - vi = a​ _
v i + vf

Next, multiply both sides by (vi + vf ) to get all the velocities on the
same side of the equation.
(vf - vi) (vf + vi) = 2a∆x = vf2 − vi2

Add vi2 to both sides to solve for vf2.

Final Velocity After Any Displacement

vf2 = vi2 + 2a∆x

(final velocity)2 = (initial velocity)2 + 2(acceleration)(displacement)

When using this equation, you must take the square root of the
right side of the equation to find the final velocity. Remember that
the square root may be either positive or negative. If you have been
consistent in your use of the sign convention, you will be able to
determine which value is the right answer by reasoning based on
the direction of the motion.

52 Chapter 2
Final Velocity After Any Displacement
Sample Problem E  A person pushing a stroller starts from
rest, uniformly accelerating at a rate of 0.500 m/s2. What is the
velocity of the stroller after it has traveled 4.75 m?

Analyze Given: vi = 0 m/s


a = 0.500 m/s2
∆x = 4.75 m
Unknown: vf = ?
Diagram:

–x +x

Choose a coordinate system. The most convenient one has an origin at


the initial location of the stroller. The positive direction is to the right.
PH99PE 002-002-010 A
Plan Choose an equation or situation:
Because the initial velocity, acceleration,
and displacement are known, the final
Tips and Tricks
velocity can be found by using the following
Think about the physical situation
equation: to determine whether to keep the
positive or negative answer from the
vf2 = vi2 + 2a∆x square root. In this case, the stroller
is speeding up because it starts
Rearrange the equation to isolate from rest and ends with a speed of
the unknown: 2.18 m/s. An object that is speeding
Take the square root of both sides up and has a positive acceleration
to isolate vf . must have a positive velocity, as
shown in Figure 2.3. So, the final
  i 
vf = ±​ √  

(v )2 + 2a∆x ​
   velocity must be positive.

solve Substitute the values into the equation and solve:


vf = ±​ √   
  2 + 2(0.500
(0 m/s)     m/s2)(4.75 m) ​

vf = +2.18 m/s

check your The stroller’s velocity after accelerating for 4.75 m is 2.18 m/s to
work the right.

Continued

Motion in One Dimension 53


Final Velocity After Any Displacement  (continued)

1.  Find the velocity after the stroller in Sample Problem E has traveled 6.32 m.
2.  A car traveling initially at +7.0 m/s accelerates uniformly at the rate of +0.80 m/s2
for a distance of 245 m.
a. What is its velocity at the end of the acceleration?
b. What is its velocity after it accelerates for 125 m?
c. What is its velocity after it accelerates for 67 m?
3.  A car accelerates uniformly in a straight line from rest at the rate of 2.3 m/s2.
a.  What is the speed of the car after it has traveled 55 m?
b.  How long does it take the car to travel 55 m?
4.  A motorboat accelerates uniformly from a velocity of 6.5 m/s to the west to a
velocity of 1.5 m/s to the west. If its acceleration was 2.7 m/s2 to the east, how far
did it travel during the acceleration?
5.  An aircraft has a liftoff speed of 33 m/s. What minimum constant acceleration
does this require if the aircraft is to be airborne after a takeoff run of 240 m?
6.  A certain car is capable of accelerating at a uniform rate of 0.85 m/s2. What is the
magnitude of the car’s displacement as it accelerates uniformly from a speed of
83 km/h to one of 94 km/h?

With the four equations presented in this section, it is possible to


solve any problem involving one-dimensional motion with uniform
acceleration. For your convenience, the equations that are used most
often are listed in Figure 2.6. The first column of the table gives the
equations in their standard form. For an object initially at rest, vi = 0.
Using this value for vi in the equations in the first column will result in
the equations in the second column. It is not necessary to memorize
the equations in the second column. If vi = 0 in any problem, you will
naturally derive this form of the equation. Referring back to the sample
problems in this chapter will guide you through using these equations
to solve many problems.

Figure 2.6
Equations for Constantly Accelerated
Straight-line Motion
Form to use when
Form to use when object
accelerating object has an
accelerating starts from rest
initial velocity

∆x = __
​ 12  ​ (vi + vf)∆t ∆x = __
​ 12 ​  vf ∆t

vf = vi + a∆t vf = a∆t

∆x = vi ∆t + __
​ 12 ​ a(∆t)2 ​ 1 ​   a(∆t)2
∆x = _
2
vf2 = vi2 + 2a∆x vf2 = 2a∆x

54 Chapter 2
Section 2 Formative ASSESSMENT
Reviewing Main Ideas
1. Marissa’s car accelerates uniformly at a rate of +2.60 m/s2. How long does
it take for Marissa’s car to accelerate from a speed of 24.6 m/s to a speed
of 26.8 m/s?
2. A bowling ball with a negative initial velocity slows down as it rolls down
the lane toward the pins. Is the bowling ball’s acceleration positive or
negative as it rolls toward the pins?
3. Nathan accelerates his skateboard uniformly along a straight path from
rest to 12.5 m/s in 2.5 s.
a. What is Nathan’s acceleration?
b. What is Nathan’s displacement during this time interval?
c. What is Nathan’s average velocity during this time interval?

Critical Thinking
4. Two cars are moving in the same direction in parallel lanes along a
highway. At some instant, the instantaneous velocity of car A exceeds the
instantaneous velocity of car B. Does this mean that car A’s acceleration is
greater than car B’s? Explain, and use examples.

Interpreting Graphics
5. The velocity-versus-time graph for a shuttle bus moving along a straight
path is shown in Figure 2.7.
a. Identify the time intervals during which the Figure 2.7
velocity of the shuttle bus is constant. Velocity Versus Time of a Shuttle Bus
b. Identify the time intervals during which the
acceleration of the shuttle bus is constant. 8.0
7.0
c. Find the value for the average velocity of 6.0
the shuttle bus during each time interval 5.0
4.0
identified in b. 3.0
Velocity (m/s)

2.0
d. Find the acceleration of the shuttle bus 1.0
during each time interval identified in b. 0
-1.0 100 200 300 400 500 600
e. Identify the times at which the velocity of the -2.0
-3.0
shuttle bus is zero. -4.0
-5.0
f. Identify the times at which the acceleration of -6.0
the shuttle bus is zero. -7.0
-8.0
g. Explain what the slope of the graph reveals Time (s)
about the acceleration in each time interval.
6. Is the shuttle bus in item 5 always moving in the TSI Graphics
same direction? Explain, and refer to the time HRW • Holt Physics
intervals shown on the graph. PH99PE-C02-002-015-A

Motion in One Dimension 55


Section 3
Objectives
Relate the motion of a freely
falling body to motion with
Falling Objects
constant acceleration.
Key Term
Calculate displacement, free fall
velocity, and time at various
points in the motion of a freely
falling object.
Free Fall
On August 2, 1971, a demonstration was conducted on the moon by
astronaut David Scott. He simultaneously released a hammer and a
feather from the same height above the moon’s surface. The hammer and
Figure 3.1 the feather both fell straight down and landed on the lunar surface at
exactly the same moment. Although the hammer is more massive than
Free Fall in a Vacuum  When
there is no air resistance, all objects the feather, both objects fell at the same rate. That is, they traveled the
fall with the same acceleration same displacement in the same amount of time.
regardless of their masses.
Freely falling bodies undergo constant acceleration.
In Figure 3.1, a feather and an apple are released from rest in a vacuum
chamber. The two objects fell at exactly the same rate, as indicated by the
horizontal alignment of the multiple images.
The amount of time that passed between the first and second images
is equal to the amount of time that passed between the fifth and sixth
images. The picture, however, shows that the displacement in each
time interval did not remain constant. Therefore, the velocity was not
constant. The apple and the feather were accelerating.
Compare the displacement between the first and second images to
the displacement between the second and third images. As you can see,
within each time interval the displacement of the feather increased by the
same amount as the displacement of the apple. Because the time intervals
are the same, we know that the velocity of each object is increasing by the
same amount in each time interval. In other words, the apple and the
feather are falling with the same constant acceleration.
If air resistance is disregarded, all objects dropped near the surface
of a planet fall with the same constant acceleration. This acceleration
is due to gravitational force, and the motion is referred to as free fall. The
acceleration due to gravity is denoted with the symbols ag (generally) or
g (on Earth’s surface). The magnitude of g is about 9.81 m/s2, or 32 ft/s2.
Unless stated otherwise, this book will use the value 9.81 m/s2 for
calculations. This acceleration is directed downward, toward the center
of Earth. In our usual choice of coordinates, the downward direction is
negative. Thus, the acceleration of objects in free fall near the surface of
free fall  the motion of a body when
©James A. Sugar/Black Star

only the force due to gravity is acting Earth is ag = −g = −9.81 m/s2. Because an object in free fall is acted on
on the body only by gravity, ag is also known as free-fall acceleration.

56 Chapter 2
Acceleration is constant during upward and FIGURE 3.2
downward motion.
Motion of a Tossed Ball At the very top
Figure 3.2 is a strobe photograph of a ball thrown up into the air
of its path, the ball’s velocity is zero, but the ball’s
with an initial upward velocity of +10.5 m/s. The photo on the
acceleration is –9.81 m/s2 at every point—both when
left shows the ball moving up from its release toward the top of it is moving up (a) and when it is moving down (b).
its path, and the photo on the right shows the ball falling back
down. Everyday experience shows that when we throw an object (a) (b)

up in the air, it will continue to move upward for some time, stop
momentarily at the peak, and then change direction and begin
to fall. Because the object changes direction, it may seem that
the velocity and acceleration are both changing. Actually,
objects thrown into the air have a downward acceleration as
soon as they are released.
In the photograph on the left, the upward displacement of
the ball between each successive image is smaller and smaller
until the ball stops and finally begins to move with an increasing
downward velocity, as shown on the right. As soon as the ball is
released with an initial upward velocity of +10.5 m/s, it has an
acceleration of −9.81 m/s2. After 1.0 s (∆t = 1.0 s), the ball’s
velocity will change by −9.81 m/s to 0.69 m/s upward. After 2.0 s
(∆t = 2.0 s), the ball’s velocity will again change by −9.81 m/s,
to −9.12 m/s.
The graph in Figure 3.3 shows the velocity of the ball
plotted against time. As you can see, there is an instant when
the velocity of the ball is equal to 0 m/s. This happens at the
instant when the ball reaches the peak of its upward motion
and is about to begin moving downward. Although the velocity
(tc) ©Richard Megna/Fundamental Photographs, New York; (tr) ©Richard Megna/Fundamental Photographs, New York

is zero at the instant the ball reaches the peak, the acceleration is
equal to −9.81 m/s2 at every instant regardless of the magnitude
or direction of the velocity. It is important to note that the
acceleration is −9.81 m/s2 even at the peak where the velocity
is zero. The straight-line slope of the graph indicates that the
acceleration is constant at every moment.

Velocity versus Time of a Tossed Ball

12
10
8
FIGURE 3.3 6
4
Slope of a Velocity-Time Graph 2
0
Velocity (m/s)

On this velocity-time graph, the slope of the -2 0.50 1.00 1.50 2.00 2.50 3.00
line, which is equal to the ball’s acceleration, is -4
constant from the moment the ball is released -6
(t = 0.00 s) and throughout its motion. -8
-10
-12
-14
-16
-18
-20
Time (s)

Motion in One Dimension


TSI Graphics
57
HRW • Holt Physics
PH99PE-C02-003-006-A
Freely falling objects always have the same downward acceleration.
It may seem a little confusing to think of something that is moving
upward, like the ball in the example, as having a downward acceleration.
Thinking of this motion as motion with a positive velocity and a negative
acceleration may help. The downward acceleration is the same when an
object is moving up, when it is at rest at the top of its path, and when it is
moving down. The only things changing are the position and the
magnitude and direction of the velocity.
When an object is thrown up in the air, it has a positive velocity and
a negative acceleration. From Figure 2.3, we see that this means the object
is slowing down as it rises in the air. From the example of the ball and
from everyday experience, we know that this makes sense. The object
continues to move upward but with a smaller and smaller speed. In the
photograph of the ball, this decrease in speed is shown by the smaller
and smaller displacements as the ball moves up to the top of its path.
At the top of its path, the object’s velocity has decreased until it is
zero. Although it is impossible to see this because it happens so quickly,
the object is actually at rest at the instant it reaches its peak position.
Even though the velocity is zero at this instant, the acceleration is
still –9.81 m/s2.
When the object begins moving down, it has a negative velocity and
its acceleration is still negative. From Figure 2.3, we see that a negative
acceleration and a negative velocity indicate an object that is speeding
up. In fact, this is what happens when objects undergo free-fall accelera-
tion. Objects that are falling toward Earth move faster and faster as they
fall. In the photograph of the ball in Figure 3.2 (on the previous page), this
increase in speed is shown by the greater and greater displacements
between the images as the ball falls.
Knowing the free-fall acceleration makes it easy to calculate the
velocity, time, and displacement of many different motions using the
equations for constantly accelerated motion. Because the acceleration is
the same throughout the entire motion, you can analyze the motion of a
freely falling object during any time interval.

Time interval OF free fall

Your reaction time affects your so that the zero mark is between from the free-fall acceleration and
performance in all kinds of your fingers with the 1 cm mark the distance the meterstick has
activities—from sports to driving above it. fallen through your grasp.
to catching something that you You should not be touching the
drop. Your reaction time is the meterstick, and your catching
time interval between an event hand must be resting on a table. Materials
and your response to it. Without warning you, your friend • meterstick or ruler
Determine your reaction time by should release the meterstick so safety
having a friend hold a meterstick that it falls between your thumb A void eye injury; do not
vertically between the thumb and and your finger. Catch the meter- swing metersticks.
index finger of your open hand. stick as quickly as you can. You
The meterstick should be held can calculate your reaction time

58 Chapter 2
Premium Content

Falling Object Interactive Demo


HMDScience.com
Sample Problem F  Jason hits a volleyball so that it moves
with an initial velocity of 6.0 m/s straight upward. If the volleyball
starts from 2.0 m above the floor, how long will it be in the air
before it strikes the floor?

Analyze Given: vi = + 6.0 m/s


a = −g = −9.81 m/s2
∆y = −2.0 m
Unknown: ∆t = ?
Diagram: Place the origin at the starting point
of the ball (yi = 0 at ti = 0).

+
y
6.0 m/s

x
2.0 m
-

Plan Choose an equation or situation:


Both ∆t and vf  are unknown. Therefore, first solve for vf  using the
equation that does not require time. Then the equation for vf  that does
involve time can be used to solve for ∆t.

vf2 = vi2 + 2a∆y   vf = vi + a∆t


Rearrange the equations to isolate the unknown:
Take the square root of the first equation to isolate vf . The second
equation must be rearranged to solve for ∆t.
vf - v i

  i2
vf = ±​ √  v    ∆t = ​ _
+ 2a∆y ​  a ​   

solve Substitute the values into the equations and solve:


First find the velocity of the ball at the moment that it hits the floor.
v  i2
vf = ±​ √     
= ±​ √

Tips and Tricks + 2a∆y ​  (6.0 m/s)2 + 2(−9.81
    m/s2)(−2.0 m) ​
When you take the square  
root to find vf , select the vf = ±​ √  
  
36 m2/s2 ​= ±​ √75
m2/s2 = 39   
  
m2/s2 ​ 
= -8.7 m/s
negative answer because
the ball will be moving Next, use this value of vf in the second equation to solve for ∆t.
toward the floor in the vf - vi -8.7 m/s - 6.0 m/s -14.7 m/s
negative direction.
∆t = ​ _ a ​  = __
​    
  
2
 ​= __
​   
 ​ 
-9.81 m/s -9.81 m/s2
∆t = 1.50 s

check your The solution, 1.50 s, is a reasonable amount of time for the ball to be in
work the air.

Continued
Motion in One Dimension 59
Falling Object  (continued)

1.  A robot probe drops a camera off the rim of a 239 m high cliff on Mars, where the
free-fall acceleration is −3.7 m/s2.
a.  Find the velocity with which the camera hits the ground.
b.  Find the time required for it to hit the ground.
2.  A flowerpot falls from a windowsill 25.0 m above the sidewalk.
a.  How fast is the flowerpot moving when it strikes the ground?
b.  How much time does a passerby on the sidewalk below have to move out of the
way before the flowerpot hits the ground?
3.  A tennis ball is thrown vertically upward with an initial velocity of +8.0 m/s.
a.  What will the ball’s speed be when it returns to its starting point?
b.  How long will the ball take to reach its starting point?
4.  Calculate the displacement of the volleyball in Sample Problem F when the
volleyball’s final velocity is 1.1 m/s upward.

Sky Diving

W
hen these sky divers jump from an airplane,
they plummet toward the ground. If Earth
had no atmosphere, the sky divers would
accelerate with the free-fall acceleration, g, equal to
9.81 m/s2. They would not slow down even after opening
their parachutes.
Fortunately, Earth does have an atmosphere, and
the acceleration of the sky divers does not remain ©Zefa Visual Media - Germany/Index Stock Imagery, Inc./Photolibrary
constant. Instead, because of air resistance, the
acceleration decreases as they fall. After a few
terminal velocity is typically about 55 m/s (123 mi/h).
seconds, the acceleration drops to zero and the speed
If the sky diver curls into a ball, the terminal velocity may
becomes constant. The constant speed an object
increase to close to 90 m/s (200 mi/h). When the sky
reaches when falling through a resisting medium is
diver opens the parachute, air resistance increases, and
called terminal velocity.
the sky diver decelerates to a new, slower terminal
The terminal velocity of an object depends on the velocity. For a sky diver with an open parachute, the
object’s mass, shape, and size. When a sky diver is terminal velocity is typically about 5 m/s (11 mi/h).
spread out horizontally to the ground, the sky diver’s

60 Chapter 2
SECTION 3 FORMATIVE ASSESSMENT
Reviewing Main Ideas
1. A coin is tossed vertically upward.
a. What happens to its velocity while it is in the air?
b. Does its acceleration increase, decrease, or remain constant while it is
in the air?
2. A pebble is dropped down a well and hits the water 1.5 s later. Using the
equations for motion with constant acceleration, determine the distance
from the edge of the well to the water’s surface.
3. A ball is thrown vertically upward. What are its velocity and acceleration
when it reaches its maximum altitude? What is its acceleration just before
it hits the ground?
4. Two children are bouncing small rubber balls. One child simply drops a
ball. At the same time, the second child throws a ball downward so that it
has an initial speed of 10 m/s. What is the acceleration of each ball while
in motion?

Critical Thinking
5. A gymnast practices two dismounts from the high bar on the uneven
parallel bars. During one dismount, she swings up off the bar with an
initial upward velocity of +4.0 m/s. In the second, she releases from the
same height but with an initial downward velocity of −3.0 m/s. What
is her acceleration in each case? How does the first final velocity as the
gymnast reaches the ground differ from the second final velocity?

Interpreting Graphics
6. Figure 3.4 is a position-time graph of the motion of a basketball thrown
straight up. Use the graph to sketch the path of the basketball and to
sketch a velocity-time graph of the basketball’s motion.

FIGURE 3.4
Position-Time Graph of a Basketball

0.5
0.0 Time (s)
–0.5 0.2 0.4 0.6 0.8 1.0 1.2 1.4
–1.0
–1.5
Position (m)

–2.0
–2.5
–3.0
–3.5
–4.0
–4.5
–5.0
–5.5

a. Is the velocityTSI
of Graphics
the basketball constant?
HRW • Holtof
b. Is the acceleration Physics
the basketball constant?
PH99PE-C02-003-015-A
c. What is the initial velocity of the basketball?

Motion in One Dimension 61


take it further

Angular Kinematics
Figure 1 A point on an object that rotates about a fixed axis undergoes circular motion
around that axis. The linear quantities introduced previously cannot be used for
Circular Motion  A light bulb on circular motion because we are considering the rotational motion of an extended
a rotating Ferris wheel (a) begins at object rather than the linear motion of a particle. For this reason, circular motion is
a point along a reference line and (b) described in terms of the change in angular position. All points on a rigid rotating
moves through an arc length s and object, except the points on the axis, move through the same angle during any
time interval.
therefore through the angle θ.

Measuring Angles with Radians


Light
bulb Many of the equations that describe circular motion require that angles be
r measured in radians (rad) rather than in degrees. To see how radians are
O Reference measured, consider Figure 1, which illustrates a light bulb on a rotating
line Ferris wheel. At t = 0, the bulb is on a fixed reference line, as shown in
Figure 1(a). After a time interval ∆t, the bulb advances to a new position, as
shown in Figure 1(b). In this time interval, the line from the center to the
(a)
bulb (depicted with a red line in both diagrams) moved through the angle θ
Light
with respect to the reference line. Likewise, the bulb moved a distance s,
bulb measured along the circumference of the circle; s is the arc length.
HRW • Holt Physics
s
PH99PE-C07-001-001-A In general, any angle θ measured in radians is defined by the following
O r Reference equation:
line arc length _s
θ=_​  = ​ r  ​ 
 ​ 

radius

(b)

Figure 2 Note that if the arc length, s, is equal to the length of the
HRW • Holt Physics radius, r, the angle θ swept by r is equal to 1 rad. Because θ is
AngularPH99PE-C07-001-002-A
Motion  Angular motion is measured in units the ratio of an arc length (a distance) to the length of the
of radians. Because there are 2π radians in a full circle,
radius (also a distance), the units cancel and the abbreviation
radians are often expressed as a multiple of π.
rad is substituted in their place. In other words, the radian is
y
a pure number, with no dimensions.
90 When the bulb on the Ferris wheel moves through an
120 1 60 angle of 360° (one revolution of the wheel), the arc length s
2π π
3 2 45
135 3
π
1
π is equal to the circumference of the circle, or 2πr.
4 3 1
150 5π π 30 Substituting this value for s into the equation above gives
6 4
1
π
the corresponding angle in radians.
6
180 π 0 θ=_​ rs  ​ = _
​ 2πr
r ​  = 2π rad
x
2π 360

6
5π 11π
330
210 4 4 7π 6
π
225 3 3π 5π 4 315
3
240 2
300
270

62 Chapter 2
Thus, 360° equals 2π rad, or one complete revolution. In other
words, one revolution corresponds to an angle of approximately
2(3.14) = 6.28 rad. Figure 2 on the previous page depicts a circle Figure 3
marked with both radians and degrees.
Angular Displacement 
It follows that any angle in degrees can be converted to an angle A light bulb on a rotating Ferris
in radians by multiplying the angle measured in degrees by 2π/360°. wheel rotates through an angular
In this way, the degrees cancel out and the measurement is left in displacement of Δθ = θ2 − θ1.
radians. The conversion relationship can be simplified as follows:
θ (rad) = _​  π   ​ θ (deg)
180°
2

Angular Displacement 1

O Reference
Just as an angle in radians is the ratio of the arc length to the radius, the line
angular displacement traveled by the bulb on the Ferris wheel is the
change in the arc length, ∆s, divided by the distance of the bulb from the
axis of rotation. This relationship is depicted in Figure 3.

Angular Displacement HRW • Holt Physics


​ ∆s
∆θ = _ r ​ 
PH99PE-C07-001-003-A

change in arc length


angular displacement (in radians) = __
​    
    ​
distance from axis

This equation is similar to the equation for linear displacement in that


this equation denotes a change in position. The difference is that this
equation gives a change in angular position rather than a change in linear
position.
For the purposes of this textbook, when a rotating object is viewed
from above, the arc length, s, is considered positive when the point
rotates counterclockwise and negative when it rotates clockwise. In other
words, ∆θ is positive when the object rotates counterclockwise and
negative when the object rotates clockwise.

radians and arc length

Use the compass to wire, and lay them the wires. Note that the Materials
draw a circle on a along the circle you angle between these • drawing compass
sheet of paper, and drew with your com- two lines equals 1 rad. • paper
mark the center point pass. Approximately How many of these • thin wire
of the circle. Measure how many pieces of angles are there in this • wire cutters or scissors
the radius of the circle, wire do you use to go circle? Repeat the safety
and cut several pieces all the way around the experiment with a  ut ends of wire
C
of wire equal to the circle? Draw lines from larger circle, and are sharp. Cut and
length of this radius. the center of the circle handle wire carefully.
compare the results of
Bend the pieces of to each end of one of each trial.

Motion in One Dimension 63


Angular Velocity
Angular velocity is defined in a manner similar to that for linear velocity.
The average angular velocity of a rotating rigid object is the ratio of the
angular displacement, ∆θ, to the corresponding time interval, ∆t. Thus,
angular velocity describes how quickly the rotation occurs. Angular
velocity is abbreviated as ωavg (ω is the Greek letter omega).

Angular Velocity
​ ∆θ ​  
ωavg = _
∆t
angular displacement
average angular velocity = ___
​       ​
time interval

Angular velocity is given in units of radians per second (rad/s).


Sometimes, angular velocities are given in revolutions per unit time.
Recall that 1 rev = 2π rad. The magnitude of angular velocity is called
angular speed.

Angular Acceleration
Figure 4 shows a bicycle turned upside down so that a repairperson can
Figure 4 work on the rear wheel. The bicycle pedals are turned so that at time t1
Angular Acceleration  the wheel has angular velocity ω1, as shown in Figure 4(a). At a later time,
An accelerating bicycle wheel rotates t2 , it has angular velocity ω2 , as shown in Figure 4(b). Because the angular
with (a) an angular velocity ω1 at time velocity is changing, there is an angular acceleration. The average angular
t1 and (b) an angular velocity ω2 at acceleration, αavg (α is the Greek letter alpha), of an object is given by the
time t2. Thus, the wheel has an angular relationship shown below. Angular acceleration has the units radians per
acceleration. second per second (rad/s2).

t1
(a) Angular Acceleration
ω - ω1 _
αavg = _
​  2 = ​ ∆ω ​ 
 ​ 
t2 - t1 ∆t
change in angular velocity
average angular acceleration = ___
​        
 ​
time interval

The relationships between the signs of angular displacement, angular


ω1 velocity, and angular acceleration are similar to those of the related linear
quantities. As discussed earlier, by convention, angular displacement is
t2 positive when an object rotates counterclockwise and negative when an
(b) object rotates clockwise. Thus, by definition, angular velocity is also
positive when an object rotates counterclockwise and negative when an
object rotates clockwise. Angular acceleration has the same sign as the
angular velocity when it increases the magnitude of the angular velocity,
and the opposite sign when it decreases the magnitude.

ω2

64 Chapter 2
take it further

If a point on the rim of a bicycle wheel had an angular velocity greater


than a point nearer the center, the shape of the wheel would be changing.
Thus, for a rotating object to remain rigid, as does a bicycle wheel or a
Ferris wheel, every portion of the object must have the same angular
velocity and the same angular acceleration. This fact is precisely what
makes angular velocity and angular acceleration so useful for describing
rotational motion.

Kinematic Equations for Constant Angular


Acceleration
All of the equations for rotational motion defined thus far are analogous
to the linear quantities defined in the chapter “Motion in One
Figure 5
Dimension.” For example, consider the following two equations:
angular substitutes
θf - θi ∆θ xf - xi ∆x
ωavg = _
​   = ​ _ ​     vavg = _
 ​  ​   = _
 ​  ​   ​   for linear quantities
tf - ti ∆t tf - ti ∆t
Angular Linear
The equations are similar, with θ replacing x and ω replacing v. The
correlations between angular and linear variables are shown in Figure 5. θ x
In light of the similarities between variables in linear motion and
ω v
those in rotational motion, it should be no surprise that the kinematic
equations of rotational motion are similar to the linear kinematic equa- α a
tions. The equations of rotational kinematics under constant angular
acceleration are summarized in Figure 6, along with the corresponding
equations for linear motion under constant acceleration. The rotational
motion equations apply only for objects rotating about a fixed axis with
constant angular acceleration.

Figure 6

rotational and linear kinematic equations


Rotational motion with Linear motion with constant
constant angular acceleration acceleration
ωf = ωi + α∆t vf = vi + a∆t

∆θ = ωi ∆t + __
​ 12 ​ α(∆t)2 ∆x = vi ∆t + __
​ 12 ​ a(∆t)2

ωf2 = ωi2 + 2α∆θ vf2 = vi2 + 2a∆x

∆θ = __
​ 12 ​  (ωi + ωf )∆t ∆x = __
​ 12 ​  (vi + vf )∆t

The quantity ω in these equations represents the instantaneous


angular velocity of the rotating object rather than the average
angular velocity.

Motion in One Dimension 65


physics on the edge

Special Relativity
and Time Dilation
While learning about kinematics, you worked with equations that describe
motion in terms of a time interval (∆t). Before Einstein developed the
special theory of relativity, everyone assumed that ∆t must be the same
for any observer, whether that observer is at rest or in motion with respect
to the event being measured. This idea is often expressed by the statement
that time is absolute.

The Relativity of Time


In 1905, Einstein challenged the assumption that time is absolute in a
paper titled “The Electrodynamics of Moving Bodies,” which contained
his special theory of relativity. The special theory of relativity applies to
Figure 1
observers and events that are moving with constant
velocity (in uniform motion) with respect to one
Measurement of Time Depends on
C02-EDG-001a-A Perspective another. One of the consequences of this theory is that
of Observer ∆t does depend on the observer’s motion.
Mirror Consider a passenger in a train that is moving
uniformly with respect to an observer standing beside
the track, as shown in Figure 1. The passenger on the
train shines a pulse of light toward a mirror directly
above him and measures the amount of time it takes
for the pulse to return. Because the passenger is
moving along with the train, he sees the pulse of light
travel directly up and then directly back down, as in
Passenger’s
Passenger’s perspective
Perspective Figure 1(a). The observer beside the track, however, sees
(a)
(a) A passenger on a train sends a pulse of light the pulse hit the mirror at an angle, as in Figure 1(b),
C02-EDG-001b-A
toward a mirror directly above. because the train is moving with respect to the track.
Thus, the distance the light travels according to the
observer is greater than the distance the light travels
from the perspective of the passenger.
One of the postulates of Einstein’s theory of
relativity, which follows from James Clerk Maxwell’s
equations about light waves, is that the speed of light is
the same for any observer, even when there is motion
between the source of light and the observer. Light is
different from all other phenomena in this respect.
(b) Observer’s
Observer’sPerspective
perspective Although this postulate seems counterintuitive, it was
(b) Relative to a stationary observer beside the strongly supported by an experiment performed in
track, the distance the light travels is greater than 1851 by Armand Fizeau. But if the speed of light is the
that measured by the passenger. same for both the passenger on the train and the

66 Chapter 2
observer beside the track while the distances traveled are different, the
time intervals observed by each person must also be different. Thus, the
observer beside the track measures a longer time interval than the
passenger does. This effect is known as time dilation.

Calculating Time Dilation


Time dilation is given by the following equation, where ∆t′ represents the
time interval measured by the person beside the track, and ∆t represents
the time interval measured by the person on the train:
​  ∆t   ​ 
∆t ′ = _
√ 
   2
​ v2 ​ ​   
​    1 - _
c
In this equation, v represents the speed of the train relative to the
person beside the track, and c is the speed of light in a vacuum,
3.00 × 108 m/s. At speeds with which we are familiar, where v is much
2
smaller than c, the term __
​ v2  ​is such a small fraction that ∆t ′ is essentially
c
equal to ∆t. For this reason, we do not observe the effects of time dilation
in our typical experiences. But when speeds are closer to the speed of
light, time dilation becomes more noticeable. As seen by this equation,
time dilation becomes infinite as v approaches the speed of light.
According to Einstein, the motion between the train and the track is
relative; that is, either system can be considered to be in motion with
respect to the other. For the passenger, the train is stationary and the
observer beside the track is in motion. If the light experiment is repeated
by the observer beside the track, then the passenger would see the light
travel a greater distance than the observer would. So, according to the
passenger, it is the observer beside the track whose clock runs more
slowly. Observers see their clocks running as if they were not moving.
Any clocks in motion relative to the observers will seem to the observers
to run slowly. Similarly, by comparing the differences between the time
intervals of their own clocks and clocks moving relative to theirs,
observers can determine how fast the other clocks are moving with
respect to their own.

Experimental Verification
The effects we have been considering hold true for all physical processes,
including chemical and biological reactions. Scientists have demon-
strated time dilation by comparing the lifetime of muons (a type of
unstable elementary particle) traveling at 0.9994c with the lifetime of
stationary muons. In another experiment, atomic clocks on jet planes
flying around the world were compared with identical clocks at the U.S.
Naval Observatory. In both cases, time dilations were observed that
matched the predictions of Einstein’s theory of special relativity within
the limits of experimental error.

Motion in One Dimension 67


careers in physics

Science Writer

S
cience writers explain science to their readers in a
clear and entertaining way. To learn more about
science writing as a career, read the interview with
Marcia Bartusiak, author of numerous books and articles
on physics and astronomy and professor of science writing
at the Massachusetts Institute of Technology.

What do you do as a science writer? What


do you write about? Bartusiak has visited the world’s largest astronomical
I specialize in writing about physics and astronomy for observatory, located on Mauna Kea Big Island, Hawaii.
popular books and magazines. When starting out in my
career, I primarily reported on new discoveries or novel
experimental techniques—on the existence of cosmic dark students to maintain a curiosity about the world and to write,
matter or the capture of neutrinos from the sun, for example. write, write. Find every opportunity to write, whether it’s for
I spend most of my time doing research and interviewing your high-school newspaper, college newspaper, or on a
scientists. If I’m writing a longer story, such as for a blog. These are ways to start flexing your writing muscles.
magazine, I often get to travel to a site. This allows me to set Science writing is all about translating complex scientific
the scene for my readers, to let people get a peek at a ideas into everyday language and then telling a story—in
laboratory or observatory that they wouldn’t otherwise have this case, the story of science. The more you do it, the better
a chance to see. you get at it.

What made you decide to become a What is the favorite part about your job?
science writer? I love the traveling. I have visited every major observatory in
When you talk to science writers, you find that we come the Northern Hemisphere. Now I’m trying for the Southern
from many different backgrounds, but we all have something Hemisphere. It’s exhilarating to watch astronomers carrying
in common: we all have an interest in both science and out their observations, then contacting

(br) ©Houghton Mifflin Harcourt; (tr) ©Technology and Industry Concepts/Alamy


writing. In college I majored in journalism and then worked them afterwards and seeing
as a reporter for four years. But I then realized that I wanted how a new universe is being
to specialize in writing about science, so I returned to school fashioned before their eyes.
and got a master’s degree in physics, the subject I loved best These are some the best
for its insights on the workings of nature. moments of my life. I get to
escape from my computer
What advice do you have to students and become acquainted
interested in writing about science? with some of the world’s top
Students don’t necessarily need to major in science in physicists and astronomers.
college. Although having a strong background in science
helps, it’s not necessary. For example, I can read papers
from scientific journals and directly recognize when an Marcia Bartusiak
important discovery is unfolding. My advice would be for

68
Chapter 2 Summary
Section 1  Displacement and Velocity Key Terms

• Displacement is a change of position in a certain direction, not the total frame of reference
distance traveled. displacement
• The average velocity of an object during some time interval is equal to the average velocity
displacement of the object divided by the time interval. Like displacement,
instantaneous velocity
velocity has both a magnitude (called speed) and a direction.
• The average velocity is equal to the slope of the straight line connecting the
initial and final points on a graph of the position of the object versus time.

Section 2  Acceleration Key Term

• The average acceleration of an object during a certain time interval is equal acceleration
to the change in the object’s velocity divided by the time interval.
Acceleration has both magnitude and direction.
• The direction of the acceleration is not always the same as the direction of
the velocity. The direction of the acceleration depends on the direction of
the motion and on whether the velocity is increasing or decreasing.
• The average acceleration is equal to the slope of the straight line
connecting the initial and final points on the graph of the velocity
of the object versus time.
• The equations in Figure 2.6 are valid whenever acceleration is constant.

Section 3  Falling Objects Key Term

• An object thrown or dropped in the presence of Earth’s gravity experiences a free fall
constant acceleration directed toward the center of Earth. This acceleration
is called the free-fall acceleration, or the acceleration due to gravity.
• Free-fall acceleration is the same for all objects, regardless of mass.
• The value for free-fall acceleration on Earth’s surface used in this book
is ag = −g = −9.81 m/s2. The direction of the free-fall acceleration is
considered to be negative because the object accelerates toward Earth.

Variable Symbols

Quantities Units

x position m meters

Δx displacement m meters

y position m meters

Δy displacement m meters Problem Solving


See Appendix D: Equations for a summary
v velocity m/s meters per second of the equations introduced in this chapter.
If you need more problem-solving practice,
a acceleration m/s2 meters per second2 see Appendix I: Additional Problems.

Chapter Summary 69
Chapter 2 Review
Displacement and Velocity Conceptual Questions
5. If the average velocity of a duck is zero in a given time
Reviewing Main Ideas
interval, what can you say about the displacement of
1. On the graph below, what is the total distance the duck for that interval?
traveled during the recorded time interval? What is
6. Velocity can be either positive or negative, depending
the displacement?
on the direction of the displacement. The time
7.0 interval, ∆t, is always positive. Why?
6.0
5.0 Practice Problems
Position (m)

4.0
For problems 7–11, see Sample Problem A.
3.0
2.0 7. A school bus takes 0.530 h to reach the school from
1.0 your house. If the average velocity of the bus is
0 19.0  km/h to the east, what is the displacement?
0 2.0 4.0 6.0 8.0 10.0
Time (s) 8. The Olympic record for the marathon is 2.00 h,
9.00 min, 21.0 s. If the average speed of a runner
2. On a position-time graph such as the one above, what achieving this record is 5.436 m/s, what is the
represents the instantaneous
TSI Graphicsvelocity? marathon distance?
HRW • Holt Physics
3. The position-time graph for a bug crawling along a
PH99PE-C02-CHS-001-A 9. Two cars are traveling on a desert road, as shown
line is shown in item 4 below. Determine whether the below. After 5.0 s, they are side by side at the next
velocity is positive, negative, or zero at each of the telephone pole. The distance between the poles is
times marked on the graph. 70.0 m. Identify the following quantities:
a. the displacement of car A after 5.0 s
4. Use the position-time graph below to answer the b. the displacement of car B after 5.0 s
following questions: c. the average velocity of car A during 5.0 s
a. During which time interval(s) is the velocity d. the average velocity of car B during 5.0 s
negative?
b. During which time interval(s) is the velocity
positive?

car A car A
Position

car B car B

0 t1 t2 t3 t4 t5
Time (a) (b)

TSI Graphics PHYSICS


Spec. Number PH 99 PE C02-CHR
HRW • Holt Physics Boston Graphics, Inc.
PH99PE-C02-CHS-004-A 617.523.1333

70 Chapter 2
Chapter review

10. Sally travels by car from one city to another. She drives 15. The diagrams below show a disk moving from left to
for 30.0 min at 80.0 km/h, 12.0 min at 105 km/h, and right under different conditions. The time interval
45.0 min at 40.0 km/h, and she spends 15.0 min eating between images is constant. Assuming that the
lunch and buying gas. direction to the right is positive, identify the following
a. Determine the average speed for the trip. types of motion in each photograph. (Some may have
b. Determine the total distance traveled. more than one type of motion.)
a. the acceleration is positive
11. Runner A is initially 6.0 km west of a flagpole and is b. the acceleration is negative
running with a constant velocity of 9.0 km/h due c. the velocity is constant
east. Runner B is initially 5.0 km east of the flagpole
and is running with a constant velocity of 8.0 km/h
due west. What will be the distance of the two
runners from the flagpole when their paths cross?
(It is not necessary to convert your answer from
kilometers to meters for this problem. You may
leave it in kilometers.)

Acceleration
Practice Problems
Reviewing Main Ideas
For problems 16–17, see Sample Problem B.
12. What would be the acceleration of a turtle that
is moving with a constant velocity of 0.25 m/s to 16. A car traveling in a straight line has a velocity of
the right? +5.0 m/s. After an acceleration of 0.75 m/s2, the car’s
velocity is +8.0 m/s. In what time interval did the
13. Sketch the velocity-time graphs for the following
acceleration occur?
motions.
a. a city bus that is moving with a constant velocity 17. The velocity-time graph for an object moving along a
b. a wheelbarrow that is speeding up at a uniform straight path is shown below. Find the average
rate of acceleration while moving in the positive accelerations during the time intervals 0.0 s to 5.0 s,
direction 5.0 s to 15.0 s, and 0.0 s to 20.0 s.
c. a tiger that is speeding up at a uniform rate of
acceleration while moving in the negative For problems 18–19, see Sample Problem C.
direction
d. an iguana that is slowing down at a uniform rate of Velocity versus Time
acceleration while moving in the positive direction 8.0
e. a camel that is slowing down at a uniform rate of 6.0
4.0
Velocity (m/s)

acceleration while moving in the negative


2.0
direction 0.0
-2.0 5.0 10.0 15.0 20.0
-4.0
Conceptual Questions -6.0
-8.0
14. If a car is traveling eastward, can its acceleration be Time (s)
westward? Explain your answer, and use an example
in your explanation.
18. A bus slows down uniformly from 75.0 km/h
(21 m/s) to 0 km/h in 21 s. How far does it travel
TSI Graphics
before stopping?
HRW • Holt Physics
PH99PE-C02-CHR-009-A

Chapter Review 71
Chapter review

19. A car accelerates uniformly from rest to a speed of 27. The image at right is a strobe photograph
65 km/h (18 m/s) in 12 s. Find the distance the car of two falling balls released simultaneously.
travels during this time. (This motion does not take place in a
vacuum.) The ball on the left side is solid,
For problems 20–23, see Sample Problem D. and the ball on the right side is a hollow
20. A car traveling at +7.0 m/s accelerates at the rate of table-tennis ball. Analyze the motion of
+0.80 m/s2 for an interval of 2.0 s. Find vf . both balls in terms of velocity and
acceleration.
21. A car accelerates from rest at −3.00 m/s2.
a. What is the velocity at the end of 5.0 s? 28. A juggler throws a bowling pin into the air
b. What is the displacement after 5.0 s? with an initial velocity vi . Another juggler
drops a pin at the same instant. Compare
22. A car starts from rest and travels for 5.0 s with a the accelerations of the two pins while they
uniform acceleration of +1.5 m/s2. The driver then are in the air.
applies the brakes, causing a uniform acceleration of
−2.0 m/s2. If the brakes are applied for 3.0 s, how fast 29. A bouquet is thrown upward.
is the car going at the end of the braking period, and a. Will the value for the bouquet’s
how far has it gone from its start? displacement be the same no matter where
you place the origin of the coordinate system?
23. A boy sledding down a hill accelerates at 1.40 m/s2. b. Will the value for the bouquet’s velocity be the
If he started from rest, in what distance would he same?
reach a speed of 7.00 m/s? c. Will the value for the bouquet’s acceleration be
the same?
For problems 24–25, see Sample Problem E.

24. A sailboat starts from rest and accelerates at a rate of Practice Problems
0.21 m/s2 over a distance of 280 m.
a. Find the magnitude of the boat’s final velocity. For problems 30–32, see Sample Problem F.
b. Find the time it takes the boat to travel this 30. A worker drops a wrench from the top of a tower
distance. 80.0 m tall. What is the velocity when the wrench
25. An elevator is moving upward at 1.20 m/s when it strikes the ground?
experiences an acceleration of 0.31 m/s2 downward, 31. A peregrine falcon dives at a pigeon. The falcon starts
over a distance of 0.75 m. What will be its final downward from rest with free-fall acceleration. If the
velocity?
pigeon is 76.0 m below the initial position of the
falcon, how long does the falcon take to reach the

Falling Objects pigeon? Assume that the pigeon remains at rest.

32. A ball is thrown upward from the ground with an


Reviewing Main Ideas initial speed of 25 m/s; at the same instant, a ball is
26. A ball is thrown vertically upward. dropped from rest from a building 15 m high. After
a. What happens to the ball’s velocity while the ball is how long will the balls be at the same height?
in the air?
b. What is its velocity when it reaches its maximum
Mixed Review
©1967 George Resch/Fundamental Photographs

altitude?
c. What is its acceleration when it reaches its maxi- Reviewing Main Ideas
mum altitude?
d. What is its acceleration just before it hits the 33. If the average speed of an orbiting space shuttle is
ground? 27 800 km/h, determine the time required for it to
e. Does its acceleration increase, decrease, or circle Earth. Assume that the shuttle is orbiting about
remain constant? 320.0 km above Earth’s surface, and that Earth’s
radius is 6380 km.

72 Chapter 2
Chapter review

34. A ball is thrown directly upward into the air. The 36. Two students are on a balcony 19.6 m above the
graph below shows the vertical position of the ball street. One student throws a ball vertically downward
with respect to time. at 14.7 m/s. At the same instant, the other student
a. How much time does the ball take to reach its throws a ball vertically upward at the same speed.
maximum height? The second ball just misses the balcony on the way
b. How much time does the ball take to reach down.
one-half its maximum height? a. What is the difference in the time the balls spend
c. Estimate the slope of ∆y/∆t at t = 0.05 s, t = 0.10 s, in the air?
t = 0.15 s, and t = 0.20 s. On your paper, draw a b. What is the velocity of each ball as it strikes the
coordinate system with velocity (v) on the y-axis ground?
and time (t) on the x-axis. Plot your velocity c. How far apart are the balls 0.800 s after they are
estimates against time. thrown?
d. From your graph, determine what the acceleration
37. A rocket moves upward, starting from rest with an
on the ball is.
acceleration of +29.4 m/s2 for 3.98 s. It runs out of
fuel at the end of the 3.98 s but does not stop. How
0.25
high does it rise above the ground?
0.20
38. Two cars travel westward along a straight highway,
Position (m)

0.15
one at a constant velocity of 85 km/h, and the other at
a constant velocity of 115 km/h.
0.10 a. Assuming that both cars start at the same point,
how much sooner does the faster car arrive at a
0.05 destination 16 km away?
b. How far must the cars travel for the faster car to
0.00
0.05 0.10 0.15 0.20 0.25 0.30 0.35 0.40 arrive 15 min before the slower car?
Time (s)
39. A small first-aid kit is dropped by a rock climber who
is descending steadily at 1.3 m/s. After 2.5 s, what is
35. A train travels between stations 1 and 2, as shown the velocity of the first-aid kit, and how far is the kit
below. The engineer of the train is instructed to start below the climber?
from rest at station 1 and accelerate uniformly
between points A and B, then coast with a uniform 40. A small fish is dropped by a pelican that is rising
velocity between points B and C, and finally acceler- steadily at 0.50 m/s.
ate uniformly between points C and D until the train a. After 2.5 s, what is the velocity of the fish?
stops at station 2. The distances AB, BC, and CD are b. How far below the pelican is the fish after 2.5 s?
all equal, and it takes 5.00 min to travel between the
41. A ranger in a national park is driving at 56 km/h when
two stations. Assume that the uniform accelerations
a deer jumps onto the road 65 m ahead of the vehicle.
have the same magnitude, even when they are
After a reaction time of t s, the ranger applies the
opposite in direction.
brakes to produce an acceleration of −3.0 m/s2. What
a. How much of this 5.00 min period does the train
is the maximum reaction time allowed if the ranger is
spend between points A and B?
to avoid hitting the deer?
b. How much of this 5.00 min period does the train
spend between points B and C?
c. How much of this 5.00 min period does the train
spend between points C and D?

Station 1 Station 2

A B C D

Chapter Review 73

PHYSICS
Chapter review

42. A speeder passes a parked police car at 30.0 m/s. 48. A model rocket is launched straight upward with
The police car starts from rest with a uniform an initial speed of 50.0 m/s. It accelerates with a
acceleration of 2.44 m/s2. constant upward acceleration of 2.00 m/s2 until
a. How much time passes before the speeder is its engines stop at an altitude of 150 m.
overtaken by the police car? a. What is the maximum height reached by the
b. How far does the speeder get before being over- rocket?
taken by the police car? b. When does the rocket reach maximum height?
c. How long is the rocket in the air?
43. An ice sled powered by a rocket engine starts from
rest on a large frozen lake and accelerates at 49. A professional racecar driver buys a car that can
+13.0 m/s2. At t1 the rocket engine is shut down and accelerate at +5.9 m/s2. The racer decides to race
the sled moves with constant velocity v until t2. against another driver in a souped-up stock car. Both
The total distance traveled by the sled is 5.30 × 103 m start from rest, but the stock-car driver leaves 1.0 s
and the total time is 90.0 s. Find t1, t2, and v. before the driver of the racecar. The stock car moves
(See Appendix A: Mathematical Review for hints on with a constant acceleration of +3.6 m/s2.
solving quadratic equations.) a. Find the time it takes the racecar driver to overtake
the stock-car driver.
44. At the 5800 m mark, the sled in the previous question b. Find the distance the two drivers travel before they
begins to accelerate at −7.0 m/s2. Use your answers are side by side.
from item 43 to answer the following questions. c. Find the velocities of both cars at the instant they
a. What is the final position of the sled when it comes are side by side.
to rest?
b. How long does it take for the sled to come to rest? 50. Two cars are traveling along a straight line in the
same direction, the lead car at 25 m/s and the other
45. A tennis ball with a velocity of +10.0 m/s to the car at 35 m/s. At the moment the cars are 45 m apart,
right is thrown perpendicularly at a wall. After the lead driver applies the brakes, causing the car to
striking the wall, the ball rebounds in the opposite have an acceleration of –2.0 m/s2.
direction with a velocity of −8.0 m/s to the left. a. How long does it take for the lead car to stop?
If the ball is in contact with the wall for 0.012 s, b. Assume that the driver of the chasing car applies
what is the average acceleration of the ball while the brakes at the same time as the driver of the
it is in contact with the wall? lead car. What must the chasing car’s minimum
46. A parachutist descending at a speed of 10.0 m/s loses negative acceleration be to avoid hitting the
a shoe at an altitude of 50.0 m. lead car?
a. When does the shoe reach the ground? c. How long does it take the chasing car to stop?
b. What is the velocity of the shoe just before it hits 51. One swimmer in a relay race has a 0.50 s lead and is
the ground? swimming at a constant speed of 4.00 m/s. The
47. A mountain climber stands at the top of a 50.0 m swimmer has 20.0 m to swim before reaching the end
cliff hanging over a calm pool of water. The climber of the pool. A second swimmer moves in the same
throws two stones vertically 1.0 s apart and direction as the leader. What constant speed must the
observes that they cause a single splash when second swimmer have in order to catch up to the
they hit the water. The first stone has an initial leader at the end of the pool?
velocity of +2.0 m/s.
a. How long after release of the first stone will the two
stones hit the water?
b. What is the initial velocity of the second stone
when it is thrown?
c. What will the velocity of each stone be at the
instant both stones hit the water?

74 Chapter 2
Chapter review

4. Research typical values for velocities and acceleration


ALTERNATIVE ASSESSMENT
of various objects. Include many examples, such as
1. Can a boat moving eastward accelerate to the west? different animals, means of transportation, sports,
What happens to the boat’s velocity? Name other continental drift, light, subatomic particles, and
examples of objects accelerating in the direction planets. Organize your findings for display on a
opposite their motion, including one with numerical poster or some other form.
values. Create diagrams and graphs. 5. Research Galileo’s work on falling bodies. What did
2. The next time you are a passenger in a car, record he want to demonstrate? What opinions or theories
the numbers displayed on the clock, the odometer, was he trying to refute? What arguments did he use to
and the speedometer every 15 s for about 5 min. persuade others that he was right? Did he depend on
Create different representations of the car’s motion, experiments, logic, findings of other scientists, or
including maps, charts, and graphs. Exchange your other approaches?
representations with someone who made a different 6. The study of various motions in nature requires
trip, and attempt to reconstruct that trip based on his devices for measuring periods of time. Prepare a
or her report. presentation on a specific type of clock, such as
3. Two stones are thrown from a cliff at the same time water clocks, sand clocks, pendulum clocks,
with the same speed, one upward and one down- wind-up clocks, atomic clocks, or biological
ward. Which stone, if either, hits the ground first? clocks. Who invented or discovered the clock?
Which, if either, hits with the higher speed? In a What scale of time does it measure? What are the
group discussion, make your best argument for each principles or phenomena behind each clock? Can
possible prediction. Set up numerical examples and they be calibrated?
solve them to test your prediction.

Motion in One Dimension


At what speed does a falling hailstone travel? Does the speed You should be able to use the table below to correlate these
depend on the distance that the hailstone falls? equations with those for an accelerating object that starts from
In this graphing calculator activity, you will have the opportu- rest.
nity to answer these questions. Your calculator will display two Motion Equations for an Object with Constant
graphs: one for displacement (distance fallen) versus time and Acceleration That Started from Rest
the other for speed versus time. These two graphs correspond
1  ​ v ∆t
∆x = ​ _
to the following two equations:
2 f 
Y1 = 4.9X2
vf = a∆t
Y2 = 9.8X
​ 1 ​   a(∆t)2
∆x = _
2
vf2 = 2a∆x

Go online to HMDScience.com to find this graphing


calculator activity.

Chapter Review 75
Standards-Based Assessment
multiple choice Use the following position-time graph of a squirrel running along a
Use the graphs below to answer questions 1–3. clothesline to answer questions 5–6.
4.0
3.0
I II

Position (m)
2.0
Position

Position
1.0
0
1.0 2.0 3.0 4.0 5.0
-1.0
Time Time
-2.0
Time (s)
III IV TSI Graphics
5. What is the squirrel’s displacement at time t = 3.0 s?
Position

Velocity

HRW • Holt Physics


A. −6.0 m PH99PE-C02-CHS-006-A
B. −2.0 m
C. +0.8 m
Time Time D. +2.0 m
1. Which graph represents an object moving with a 6. What is the squirrel’s average velocity during the
constant positive velocity? time interval between 0.0 s and 3.0 s?
A. I F. −2.0 m/s
B. II G. −0.67 m/s
C. III H. 0.0 m/s
D. IV J. +0.53 m/s
2. Which graph represents an object at rest? 7. Which of the following statements is true of
F. I acceleration?
G. II A. Acceleration always has the same sign as
H. III displacement.
J. IV B. Acceleration always has the same sign as velocity.
3. Which graph represents an object moving with C. The sign of acceleration depends on both
constant positive acceleration? the direction of motion and how the velocity
A. I is changing.
B. II D. Acceleration always has a positive sign.
C. III 8. A ball initially at rest rolls down a hill and has an
D. IV acceleration of 3.3 m/s2. If it accelerates for 7.5 s,
4. A bus travels from El Paso, Texas, to Chihuahua, how far will it move during this time?
Mexico, in 5.2 h with an average velocity of 73 km/h F. 12 m
to the south. What is the bus’s displacement? G. 93 m
F. 73 km to the south H. 120 m
G. 370 km to the south J. 190 m
H. 380 km to the south
J. 14 km/h to the south

76 Chapter 2
Test Prep

9. Which of the following statements is true for a ball 12. For an object moving with constant negative
thrown vertically upward? acceleration, draw the following:
A. The ball has a negative acceleration on the way a. a graph of position versus time
up and a positive acceleration on the way down. b. a graph of velocity versus time
B. The ball has a positive acceleration on the way up
For both graphs, assume the object starts with a
and a negative acceleration on the way down.
positive velocity and a positive displacement from
C. The ball has zero acceleration on the way up and
a positive acceleration on the way down. the origin.
D. The ball has a constant acceleration throughout 13. A snowmobile travels in a straight line. The snow-
its flight. mobile’s initial velocity is +3.0 m/s.
a. If the snowmobile accelerates at a rate of
SHORT RESPONSE +0.50 m/s2 for 7.0 s, what is its final velocity?
b. If the snowmobile accelerates at the rate of
10. In one or two sentences, explain the difference −0.60 m/s2 from its initial velocity of +3.0 m/s,
between displacement and distance traveled. how long will it take to reach a complete stop?
11. The graph below shows the position of a runner at
different times during a run. Use the graph to EXTENDED RESPONSE
determine the runner’s displacement and average
velocity: 14. A car moving eastward along a straight road
a. for the time interval from t = 0.0 min to increases its speed uniformly from 16 m/s to 32 m/s
t = 10.0 min in 10.0 s.
b. for the time interval from t = 10.0 min to a. What is the car’s average acceleration?
t = 20.0 min b. What is the car’s average velocity?
c. for the time interval from t = 20.0 min to c. How far did the car move while accelerating?
t = 30.0 min Show all of your work for these calculations.
d. for the entire run
15. A ball is thrown vertically upward with a speed of
25.0 m/s from a height of 2.0 m.
5.0
a. How long does it take the ball to reach its highest
point?
4.0 b. How long is the ball in the air?
Show all of your work for these calculations.
Position ( 10³m)

3.0

2.0

1.0

0.0
0.0 10.0 20.0 30.0 40.0
Time (min)

11 12 1 Test Tip
10 2
9 3 When filling in your answers on an
8
5
4 answer sheet, always check to make
7 6
sure you are filling in the answer for the
right question. If you have to change
an answer, be sure to completely erase
your previous answer.

Standards-Based Assessment 77

You might also like